Difference between revisions of "1971 AHSME Problems/Problem 29"

(Created page with "== Problem 29 == Given the progression <math>10^{\dfrac{1}{11}}, 10^{\dfrac{2}{11}}, 10^{\dfrac{3}{11}}, 10^{\dfrac{4}{11}},\dots , 10^{\dfrac{n}{11}}</math>. The least posi...")
 
(Problem 29)
Line 9: Line 9:
 
\textbf{(D) }10\qquad  
 
\textbf{(D) }10\qquad  
 
\textbf{(E) }11    </math>
 
\textbf{(E) }11    </math>
 
[[1971 AHSME Problems/Problem 29|Solution]]
 
  
 
==Solution==
 
==Solution==

Revision as of 18:02, 22 August 2019

Problem 29

Given the progression $10^{\dfrac{1}{11}}, 10^{\dfrac{2}{11}}, 10^{\dfrac{3}{11}}, 10^{\dfrac{4}{11}},\dots , 10^{\dfrac{n}{11}}$. The least positive integer $n$ such that the product of the first $n$ terms of the progression exceeds $100,000$ is

$\textbf{(A) }7\qquad \textbf{(B) }8\qquad \textbf{(C) }9\qquad \textbf{(D) }10\qquad  \textbf{(E) }11$

Solution